Đến nội dung

Hình ảnh

$(2^{n}+1)\vdots n^{2}$

- - - - - số học

  • Please log in to reply
Chủ đề này có 5 trả lời

#1
Dam Uoc Mo

Dam Uoc Mo

    Sĩ quan

  • Thành viên
  • 433 Bài viết

Cho $n=3.l (l\in N^{*}),l\equiv 1(mod2),(l,3)=1$.

Tìm các giá trị của $l$ để $(2^{n}+1)\vdots n^{2}$.


Batman: Anh hùng có thể là bất kì ai. Thậm chí là một người đàn ông với một hành động đơn giản như đặt lên vai một cậu bé chiếc áo khoác một cách an toàn, để cho cậu ấy biết rằng thế giới vẫn chưa đi tới hồi kết. – The Dark Knight Rises.

 

 

http://news.go.vn/di...m-nguoi-doi.htm


#2
I Love MC

I Love MC

    Đại úy

  • Thành viên nổi bật 2016
  • 1861 Bài viết

Đầu tiên ta nhắc lại bổ đề quen thuộc sau : 
Bổ đề (*) : Với $m$ là số nguyên dương và $gcd(a,m)=gcd(b,m)=1$ thì lúc đó nếu $a^x \equiv b^x \pmod{m} ,a^y \equiv b^y \pmod{m}$ 
Thì $a^{gcd(x,y)} \equiv b^{gcd(x,y)} \pmod{m}$ 
Có thể xem cách chứng minh tại đây 
Bổ đề : Nếu $n|2^n+1$ thì $n=3^q.i$   với $(i,3)=1$
Chứng minh : $n$ lẻ đó là điều ta nhận thấy . Gọi $p$ là ước nguyên tố nhỏ nhất của $n$ ta có từ FLT  : 
$2^{p-1} \equiv 1 \pmod{p}$ và $2^{2n} \equiv 1 \pmod{p}$ suy ra $2^{gcd(p-1,2n)} \equiv 1 \pmod{p}$ 
Mà  dễ thấy $gcd(p-1,2n)=2$ (do $p$ là ước nguyên tố nhỏ nhất của $n$ ) 
Do vậy $p=3$ 
Suy ra $n=3^q.i$ với $(i,3)=1$
Quay về bài toán :Xét $n=1$ thì thỏa với $n>1$ thì 
Bằng LTE ta dễ suy ra được $q=1$ do đó $n=3.i$
Gọi $m$ là ước nguyên tố nhỏ nhất của $i$ khi đó : $m|2^{gcd(m-1,2n)}-1$ mà ta có $gcd(m-1,2n)$ nhận giá trị $2$ hay $6$ nếu là $2$ thì $m=3$ 
đây là điều vô lí . Nếu là $6$ thì ta suy ra $m=7$ từ đó suy ra $7|2^n+1$ ,vô lí vì $2^n+1 \equiv 2 \pmod{7}$ do đó $i=1$ hay $n=3$ 
P/s : Thực ra bài này ko  cần những điều kiện trên vẫn giải được ,bạn có thể tham khảo lời giải và có thể lược bỏ một số ý mà bạn cho là không cần thiết



#3
hoangvunamtan123

hoangvunamtan123

    Trung sĩ

  • Banned
  • 107 Bài viết

cái này là đề (IMO 1990 )

theo đề thì n chia hết cho 3 

đặt v3(n)=k Ta có  v3(2n+1) =v3(3) + k =1+k

ma 2n+1 chia hết  cho n2 nên  v3(2n+1)$\geq$ v3(n2) nen 1+k$\geq k$ nen k=1

goi q :ước nguyên tố nhỏ nhất của  x

đặt  n=3x ta có  26x$\equiv 1(mod q)$ chọn l là số nhỏ nhất  sao cho 2l$\equiv 1 ( mod q )$ nên  6x chia hết cho  l 

suy ra l = 3:6 

xét l=3 suy ra q=7 tương tự với l=6 thi 63 chia hết  cho l vì vậy l =7.lúc này sẽ mâu thuẫn vi : 26x+1$\equiv 2(mod 7 )$ va n2  chia hết cho 7 

do n chia hết cho 3 nên n =3 .

vậy :n= 3 thõa yêu cầu bài toán 


Bài viết đã được chỉnh sửa nội dung bởi hoangvunamtan123: 07-09-2016 - 18:47


#4
Dam Uoc Mo

Dam Uoc Mo

    Sĩ quan

  • Thành viên
  • 433 Bài viết

Đầu tiên ta nhắc lại bổ đề quen thuộc sau : 
Bổ đề (*) : Với $m$ là số nguyên dương và $gcd(a,m)=gcd(b,m)=1$ thì lúc đó nếu $a^x \equiv b^x \pmod{m} ,a^y \equiv b^y \pmod{m}$ 
Thì $a^{gcd(x,y)} \equiv b^{gcd(x,y)} \pmod{m}$ 
Có thể xem cách chứng minh tại đây 
Bổ đề : Nếu $n|2^n+1$ thì $n=3^q.i$   với $(i,3)=1$
Chứng minh : $n$ lẻ đó là điều ta nhận thấy . Gọi $p$ là ước nguyên tố nhỏ nhất của $n$ ta có từ FLT  : 
$2^{p-1} \equiv 1 \pmod{p}$ và $2^{2n} \equiv 1 \pmod{p}$ suy ra $2^{gcd(p-1,2n)} \equiv 1 \pmod{p}$ 
Mà  dễ thấy $gcd(p-1,2n)=2$ (do $p$ là ước nguyên tố nhỏ nhất của $n$ ) 
Do vậy $p=3$ 
Suy ra $n=3^q.i$ với $(i,3)=1$
Quay về bài toán :Xét $n=1$ thì thỏa với $n>1$ thì 
Bằng LTE ta dễ suy ra được $q=1$ do đó $n=3.i$
Gọi $m$ là ước nguyên tố nhỏ nhất của $i$ khi đó : $m|2^{gcd(m-1,2n)}-1$ mà ta có $gcd(m-1,2n)$ nhận giá trị $2$ hay $6$ nếu là $2$ thì $m=3$ 
đây là điều vô lí . Nếu là $6$ thì ta suy ra $m=7$
từ đó suy ra $7|2^n+1$ ,vô lí vì $2^n+1 \equiv 2 \pmod{7}$ do đó $i=1$ hay $n=3$ 
P/s : Thực ra bài này ko  cần những điều kiện trên vẫn giải được ,bạn có thể tham khảo lời giải và có thể lược bỏ một số ý mà bạn cho là không cần thiết

làm rõ giúp mình đoạn này với. 

 

cái này là đề (IMO 1990 )

theo đề thì n chia hết cho 3 

đặt v3(n)=k Ta có  v3(2n+1) =v3(3) + k =1+k

ma 2n+1 chia hết  cho n2 nên  v3(2n+1)$\geq$ v3(n2) nen 1+k$\geq k$ nen k=1

goi q :ước nguyên tố nhỏ nhất của  x

đặt  n=3x ta có  26x$\equiv 1(mod q)$ chọn l là số nhỏ nhất  sao cho 2l$\equiv 1 ( mod q )$ nên  6x chia hết cho  l 

suy ra l = 3:6 

xét l=3 suy ra q=7 tương tự với l=6 thi 63 chia hết  cho l vì vậy l =7.lúc này sẽ mâu thuẫn vi : 26x+1$\equiv 2(mod 7 )$ va n2  chia hết cho 7 

do n chia hết cho 3 nên n =3 .

vậy :n= 3 thõa yêu cầu bài toán 

làm rõ giúp mình đoạn này với.

Đây đúng là 1 bài toán mở rộng IMO. Mình chứng minh được đến kết quả kia thì bị nghẽn nên nhờ mọi người làm nốt, phần dữ kiện thừa là do mình cung cấp từ những gì mình đã có cho m.n đỡ phải c.m ấy mà. :D


Batman: Anh hùng có thể là bất kì ai. Thậm chí là một người đàn ông với một hành động đơn giản như đặt lên vai một cậu bé chiếc áo khoác một cách an toàn, để cho cậu ấy biết rằng thế giới vẫn chưa đi tới hồi kết. – The Dark Knight Rises.

 

 

http://news.go.vn/di...m-nguoi-doi.htm


#5
Dam Uoc Mo

Dam Uoc Mo

    Sĩ quan

  • Thành viên
  • 433 Bài viết

Mình nghĩ 2 cách giải trên đã có nhầm lẫn, mình sẽ tiếp tục ý tưởng như này: $2^{n}+1\equiv 0(mod n^{2})\\ --> (-8)^{l}\equiv 1(mod 9l^{2})\\ -->(-8)^{l}\equiv 1(modl)$. Đến đây có thể gọi m là ước nguyên tố bé nhất của l rồi dùng bậc của số nguyên để giải quyết nốt. :D


Batman: Anh hùng có thể là bất kì ai. Thậm chí là một người đàn ông với một hành động đơn giản như đặt lên vai một cậu bé chiếc áo khoác một cách an toàn, để cho cậu ấy biết rằng thế giới vẫn chưa đi tới hồi kết. – The Dark Knight Rises.

 

 

http://news.go.vn/di...m-nguoi-doi.htm


#6
I Love MC

I Love MC

    Đại úy

  • Thành viên nổi bật 2016
  • 1861 Bài viết

Mình nghĩ 2 cách giải trên đã có nhầm lẫn, mình sẽ tiếp tục ý tưởng như này: $2^{n}+1\equiv 0(mod n^{2})\\ --> (-8)^{l}\equiv 1(mod 9l^{2})\\ -->(-8)^{l}\equiv 1(modl)$. Đến đây có thể gọi m là ước nguyên tố bé nhất của l rồi dùng bậc của số nguyên để giải quyết nốt. :D

Nếu bạn muốn cách khác : 
Bắt đầu từ đoạn ta có $n=3s$ với $(s,3)=1$ 
Gọi $q$ là ước nguyên tố nhỏ nhất của  $s$  ,vì $(s,3)=1$ nên $q>5$  . Đặt $k=ord_q(2)$ ta có $2^{2n} \equiv 1 \pmod{q}$ dẫn đến $k|6s=2n$ và $k|q-1$ hay $k|(6s,q-1)$ mà $(s,q-1)=1$ nên $k=2$ suy ra $3 \vdots q$ với $q>5$ ,vô lí . Suy ra $s=1$ 







Được gắn nhãn với một hoặc nhiều trong số những từ khóa sau: số học

0 người đang xem chủ đề

0 thành viên, 0 khách, 0 thành viên ẩn danh